The Fast Track Summer 14

Page 1

Get outdoors and enjoy the sunshine!

The Fast Track Summer 2014 - Issue 11

AN EMERGENCY MEDICINE PUBLICATION

Spring 2014

The Fast Track

ACOEP’s Response to the AOA-ACGME Merger

Wilderness Medicine Special Edition

Stabilize a Broken Bone

Snake Bites! Drowning: What You Need to Know For Summer

page 1


The Track TheFast Fast Track Editors & Publishers Spring 2014

Tanner Gronowski Drew Kalnow Joe Sorber Andrew Little Jeremy Lacocque Giles Gifford Danielle Turrin Kaitlin Fries Erin Sernoffsky

Issue Contributors Tanner Gronowski Steve Brandon Kaitlin Fries Chase Ungs Danielle Turrin Brian Lehnhof Judd Shelton Mark Mitchell Alexandra Murray Lisa Hrutkay Ben Abo Stephen Vetrano Geoffery Comp Veronica Coopersmith Julie Littwin Sasha Rihter

Interested in contributing? Let us know: FastTrack@ACOEP.org

Printing of this issue sponsored by:

Letter from the Editor

A

ll of us - medical students and residents alike - are in a career path that constantly pushes us to the next level. When we think that there is no possible way we can handle one more single responsibility or time consuming activity, our programs find a way to sneak it in there. Putting board studying on top of classroom studying. Adding clinical rotations. Learning to be an intern. Learning to teach as a senior resident. Every year there seem to be new goals and responsibilities in addition to everything we already do. Often times, the mere thought of the future can be overwhelming... But that is what makes us who we are. How do you react when the bar is set just a little higher? Do you fight, climb, and claw your way to next level? And when you get there, are you content to sit with your hands on your knees? If there is one message I have learned in the past year as an intern and ACOEP Resident Chapter board member, it is to never be content with where you are sitting. Putting in the effort to get to the next level is important but once you get there and have a moment to catch your breath, continue to look higher. Here at the Fast Track we are attempting to do the same thing. We are constantly trying to raise that bar, one issue at a time. One of our biggest accomplishments in recent memory occurred in the previous issue, when we got to physically hold a printed version of the ACOEP publication and hand it out to the readers at the Spring Conference. Most importantly, however, it could not be done without you, the readers. If it weren’t for those of you who read the publication, write and contribute articles, and give us a higher bar to reach for, we would never succeed. Please enjoy this special wilderness medicine issue, as the topics are near and dear to my heart personally as an outdoor enthusiast. Keep your chin up, continue to climb to the next level, and keep looking higher.

Tanner Gronowski, DO Doctors Hospital Emergency Medicine Residency

CONTENTS

Presidential Messages .................................... 04 By Kaitlin Fries and Steve Brandon

Rosh Review........................................................ 06 page 2 photo courtesy of Tanner Gronowski Cover

Emergency Medicine Board Style Questions


Spring 2014

The Fast Track

DROWNING by Ben Abo

20

SNAKE BITES By Nate Moore

Pain Posters ......................................................... 08 By Judd Shelton

Tricks of the Trade ........................................... 09 By Chase Ungs

09

TRICKS OF THE TRADE by Chase Ungs

Pimpology ............................................................ 10 By Danielle Turrin

AOA/ACGME Merger....................................... 12 By Mark Mitchell

Therapeutic Hypothermia ........................... 14 By Alexandra Murray

Renewing Your Passion ................................. 23

WILDERNESS MED Special Section!

By Lisa Hrutkay

Drowning .............................................................. 32 By Ben Abo

Truly Lifesaving Skills of EMS...................... 35 By Stephen J. Vetrano

View from the Bottom ................................... 38 By Brian Lenhoff

RESIDENCY SPOTLIGHT OVMC

page 3


The Fast Track

PRESIDENTIAL MESSAGE – Spring 2014

Resident Chapter uly is a time of transition for those of us on the long road of training to become a physician. Many are starting new jobs or residencies, the rest of us moving up a year in our training and taking on more responsibility. While these are good, we approach these changes with trepidation and hesitation. I want to touch on a change that will be affecting us all in the not too distant future, which is the merging of osteopathic and allopathic training programs. I talked about this in my most recent article in the Pulse, but I feel it is important enough to warrant touching on again.

J

Like all change, there is fear associated with this merger. And I understand this. It is important to first remember that this is a change that had to happen. Without this, DOs coming out of medical school would have had much fewer options for training. While some worry about this merger causing us to “lose our DO identity”, the fact is that losing out on allopathic training options would have been detrimental to the future of Osteopathic Medicine. That being said, we still don’t have all of the information about the details of how this will all happen. Much of this is still being ironed out. Realize that the announcement of the merger four months ago was just that, an announcement. They were letting us know of their intent to merge, but admitted that there was still much to be figured out. I do wish we had more information but appreciate being appraised of developments as they come out. I encourage you to each check out the AOA’s website, I was actually surprised by how much information was on there. I am not worried. One of the things the AOA is best at is looking out for and fighting for osteopathic medicine. I am confident that they will not make decisions that could potentially jeopardize their future and ours. What we can do now is get involved and active where we can. The ACOEP has taken a very active role in advocating for us as this change approaches. We have a lot of people in our corner and we can take solace in this. I am confident that we will come out of this transition better and stronger than ever.

Sincerely, Steve Brandon, DO St. Mary Mercy Hospital

page 4


PRESIDENTIAL MESSAGE –

Student Chapter

Spring 2014

The Fast Track

W

elcome to another great edition of the Fast Track! This is an exciting time for all of us as we transition into a new school year. Your student chapter board is hard at work planning another fun-filled Fall conference. Mark your calendars for October 11th - 13th at Caesar’s Palace in Las Vegas, NV. The student hotel deal has already been announced so there is nothing stopping you from booking your trip today! We have tons of great things in store for you including our rapid-fire lecture series, skills labs, and the Student Residency Expo. For our third- and fourth-year students, the mock interviews are back to provide you with feedback for your upcoming residency interviews. Keep an eye out for information regarding sign-ups. Another exciting part of fall conference is finding students who are eager to join our student chapter board! We will be holding elections for our four executive and seven board member positions in Las Vegas this fall. If you are interested in getting more involved this is a great opportunity. There is far more to gain from holding an elected position then just a line on your résumé. The leadership skills, friendships, networking, and insight into the field of emergency medicine that I have gained throughout my time on the board are priceless. Even if you are not completely sure emergency medicine is for you, there is no better way to find out. More information will be sent out regarding elections within the next few months. However, if you want to know more about any of the elected positions, please do not hesitate to contact any of our board members. We would love to hear from you. We hope to see you all in Vegas!

Sincerely, Kaitlin Fries, OMS-IV OU-HCOM

page 5


The T he Fast Fast Track Track

Spring 2014

Emergency Medicine Review with 1. An 18-month-old boy is brought in by his parents for shortness of breath. The parents woke to him coughing a high-pitched cough. They also noted other noises when he was breathing in that resolved upon walking outside. The patient is frequently coughing but has no abnormal sounds on auscultation of the neck or lungs. What is the most appropriate treatment? A. Albuterol B. Dexamethasone C. Racemic epinephrine D. Ribavirin

Find more questions like these by visiting roshreview.com

2. A 4-year-old boy without vaccinations presents with a sore throat. Mom has also noted that the patient’s voice appears more muffled. On physical examination, the patient is ill appearing, sitting forward with some drooling. What is the most appropriate next step?

page 6

A. Insert two intravenous lines B. Inspect the posterior pharynx using a tongue depressor C. Move the patient to the operating room for evaluation with ENT D. Racemic epinephrine nebulizer 3. An 18-month-old infant is brought in by parents after he rolled off the changing table. He is acting normally but has a scalp hematoma. Which of the following locations of a scalp hematoma in an otherwise healthy child negates the need for a CT scan? A. Forehead B. Occipital bone C. Parietal bone D. Temporal bone 4. In an emergency medical services system, which of the following has the highest level of training and advanced practice? A. Advanced emergency medical technician (AEMT) B. Emergency medical dispatcher (EMD) C. Emergency medical technician (EMT) D. Emergency medical technician – Paramedic (EMT-P)

Find your Rosh Review Answers on page 46


The Fast Track

Spring 2014

The Edge: Scientific Assembly 2014

Student and Resident Conference Caesars Palace • Las Vegas, NV October 11th - 13th Residency Fair High Impact Lectures and Labs Evening Events Networking For more information and to regester www.acoep.org Book a discounted room at Bally’s https://resweb.passkey.com/go/SBAMC4

page pa p age g 7


The Fast Track

Spring 2014

Pain Posters by Judd Shelton, OMS-IV @ WVSOM

T

here has been a lot of media attention as of late regarding the abuse of narcotic medications. Just this past March there was an article published on the front page of of a leading newspaper that was titled “Doctors are Top Source of Prescription Drugs for Chronic Abusers.” With the increase in attention and a target placed squarely on doctors many physicians are trying to curb the trend by being more selective in whom pain medications are prescribed. The emergency department is on the front lines, it is where patients turn when they have acute problems, breakthrough pain, those looking to obtain medication for chronic pain or recreational use. One deterrent that many emergency departments have utilized to try and discourage patients seeking pain management for chronic conditions or recreational use is the use of posters prominently displayed in waiting rooms that state that medications such as OxyContin, Methadone, Suboxone etc. will not be prescribed in the emergency department. As well intentioned as these posters are their days appear to be numbered. Last year the South Carolina Hospital Association requested an opinion from their regional office of the Centers for Medicare & Medicaid Services (CMS) regarding the legality of these posters with regard to the Emergency Medical Treatment and Active Labor Act (EMTALA). EMTALA is a federal law that was passed in 1986 which applies to hospitals that receive payment from Medicare. With respect to emergency departments, the law mandates that anyone seeking medical attention regardless of their ability to pay must receive a medical screening exam. CMS has ten regional offices across the country and is responsible for the enforcement of EMTALA. The enforcement of EMTALA is through a complaint driven system so when a complaint is filed an investigation is opened. If a complaint against a hospital or physician is deemed to be in violation of EMTALA the physician and/or Hospital can be fined up to $50,000 per violation. In reply to the request by the South Carolina Hospital Association, Dr. Rick Wild the CMS Chief Medical Officer for the Atlanta Regional Office(region 4) issued an opinion that strongly discouraged the use of prescription pain posters or signage of similar content. Some have speculated that since this was just one person’s opinion it only applies to CMS Region 4. Dr. Wild however, clarifies these speculations this past January. Dr. Wild affirms that his opinion was based on the wording of the law where in an “emergency medical condition” is defined as “ a medical condition manifesting itself by acute symptoms of sufficient severity (including severe pain)…” The concern is that these posters might discourage patients from seeking a medical screening exam based on their presentation of pain and would therefore be in violation of EMTALA. Dr. Wild goes on to state that the opinion was drafted with input from the central CMS office and that his ruling was based on the current standards and policies surrounding EMTALA and not on a new interpretation of the law. Dr Wild expects that given the response to his opinion, the Central CMS office may issue a memo to all regional offices on the topic. So what does this mean for emergency departments? Will many have to change their practices? Likely the only change that will happen is that the signs will probably come down. Physicians will continue to give all patients presenting to the emergency department a medical screening exam to determine if they have an emergency medical condition. The opinion does not state that physicians must give a specific medication to patients, it merely cautions against the suggestion of available care being withheld from patients based on their presenting symptoms. +

page 8


Tricks of the Trade By Chase Ungs, DO @ Doctors Hospital

Spring 2014

The Fast Track

Fractures in the WILD 1. Perform the ABCs of trauma. 2. Assess neurovascular status of the extremity by palpating pulses, evaluating for sensory deďŹ cits, and evaluating cap reďŹ ll. 3. If neurovascular compromise is present, reduction must be performed with reevaluation of neurovascular status after reduction and placement of the splint. 4. Apply ice (if available) on wound to prevent edema and decrease pain. Oral pain medications may be required such as acetaminophen or ibuprofen . 5. If open fracture and extensive uncontrolled bleeding, a pressure dressing or tourniquet may be required. Be aware that applying an unnecessary tourniquet may result in vascular compromise if applied for an extended period of time. 6. Irrigate wound with clean water. 7. If no neurovascular compromise, do not attempt any reduction. Splint the injury with anything found. Items that may be used include paddle oars, aluminum cooking pans, life jacket, twigs/sticks, or a pillow. Ensure you have string, tape or something else to wrap the splint to keep in place. +

page 9


Pimpology 101 The Fast Track

Spring 2014

Top Things to Know on your EM Rotations Danielle Turrin, DO, MS @ Good Samaritan Hospital

The first step in handling dental emergencies is knowing the language and anatomy of the mouth. There is a Universal Numbering System used by dental professionals so they have a language and a way to understand one another. Adults typically have 32 teeth which are numbered from top right as #1 through top left as #16 and wrap around from bottom left as #17 to bottom right as #32. The numbering system is modified for primary teeth or “baby teeth” which are lettered A-T in the same fashion beginning top right and ending bottom right. Dental Anatomy The next important thing to know in dental emergencies is the anatomy of the tooth itself. The outer layer of the tooth is called the enamel layer, it is the mineralized outer layer that protects the tooth from erosion and is white in color. Due to the mineralization, enamel is hard, yet brittle. The middle layer is the dentin that forms the supporting framework for the tooth and is generally yellow in color. Dentin is not quite as strong as enamel, but is much less brittle. The inner layer is the pulp layer that contains the nerves and blood vessels, and when exposed is pink in color. The pulp layer is extremely sensitive and prone to infection if exposed.

Ellis Fracture Classification & Treatment Ellis I: Includes crown fractures that extend through the enamel only. These teeth are usually nontender and without visible color change but have rough edges. Tx: File down sharp edges, refer to dentist for cosmetic reasons only. Ellis II: Involve enamel as well as the dentin layer. These teeth are typically tender to the touch and to air exposure. A yellow layer of dentin may be visible on examination. Tx: Cover exposed surface with dental composite or Dermabond if no composite available, analgesia, avoidance of hot & cold foods/beverages, dental follow up in 24 hours, consider antibiotics (Clindamycin or Penicillin) Ellis III: Involve all three layers, are also tender and have a visible area of pink or possibly even blood at tooth’s center. Tx: Cover exposed surface with dental composite or Dermabond if no composite available, analgesia, immediate dental follow up, initiate antibiotics as exposed pulp is prone to infection (Clindamycin or Penicillin) page 10

For more quick, easy, factual pimp-question information don’t forget to check our Pimpology 101 in our next issue of the Fast Track...


The Annual FOEM Research Competitions

Spring 2014

The Fast Track

Scientific Assembly of the ACOEP in October 2014 at Ceasar’s Palace Resort in Las Vegas FOEM Research Study Poster Competition

FOEM Oral Abstract Competition

Open to residents and students that have completed a research project and would like to present it as a poster summarizing their findings.

Open to those whom have completed a research project and would like to present it as a PowerPoint presentation.

FOEM Clinical Pathological Case Competition (CPC)

FOEM Resident Research Paper Competition

This exciting annual competition pits residents against faculty in diagnosing a difficult case. Residents submit the case without final diagnosis, and the faculty member is given one month to develop a diagnosis. Both residents and faculty submit PowerPoint presentations.

This is FOEM’s most prestigious event. Participants submit their research papers (already required to complete residency training) for review. The top 3 winning resident-authors present their findings at the ACOEP Scientific Assembly annually.

Boost your CV!

Application deadline for all competitions is July 31, 2014

Win Money!

GO TO WWW.FOEM.ORG FOR MORE INFORMATION

page 11


The Fast Track

Spring 2014

AOA/ACGME MERGER By Mark Mitchell, D.O., FACOEP, FACEP President, ACOEP

Yet again

it is another season of change and one of the next changes involves the accreditation of Graduate Medical Education(GME). I am sure that you are all well aware of the Memorandum of Understanding (MOU) that was entered into by the American Osteopathic Association (AOA), American Association of Colleges of Osteopathic Medicine (AACOM), and the Accreditation Council for Graduate Medical Education (ACGME) in February, 2014. This was the culmination of several years of work to get to this point. Looking at the “big picture”, twenty percent (20%) of all graduates of Medical Education in the US are Osteopathic Physicians. However, there is a major gap in the number of graduates compared to the number of Osteopathic residency positions available. Currently there are over 3,000 more graduates than there are Osteopathic GME positions. Therefore, some change was needed in order to insure that the graduating osteopathic medical students had as many options for GME as possible. Also, the ACGME issued new standards that would have dramatically restricted Osteopathic medical school graduates if a single common pathway was not created. It would have not allowed graduates of Osteopathic residency programs to qualify for ACGME Fellowships, among other things. The process leading up the signing of the MOU was a very labor intensive one that took a couple of years. At the AOA House of Delegates in 2013 a vote was cast at that time NOT to enter into the MOU as it was presented at that time. This was due to some language that was not acceptable in the original MOU. This did not signal the end of dialog, but conversely an increase in intensity to insure that all parties involved felt comfortable with the direction. At this point there are still many unanswered questions about the details of how this will eventually work. One of the key issues that we are addressing is to insure that our current Program Directors credentials are accepted and they will be able to continue to provide outstanding leadership. However, I have the utmost confidence that the leadership of the AOA will continue to work to insure that every Osteopathic medical student has the opportunity to get the best training available. Many of us are also looking

page 12


for clarity around the certification process. We want to insure that all graduates of GME programs with an Osteopathic focus take the Osteopathic certifying boards.

Spring 2014

The Fast Track

The leadership of ACOEP continues to meet and have discussions on the potential impact this will have to our organization. There are some obvious financial implications that we can easily overcome. The biggest issues we are dealing with is to insure that we continue to be a “value added” organization for our membership. Our CME programs continue to improve and the attendance at these continue to break records. We are working closely with ACEP by combining our voices we have an even bigger impact on topics such as federal legislation. Our Osteopathic emergency medicine residency programs are very strong and continue to grow. Our graduates are high quality physicians that are highly sought after when then complete their training. Graduates of our programs have gone on to become leaders in many different venues. Some are even teaching facility at ACGME emergency medicine programs. I have had many interactions with other emergency medicine leaders including Alex Rosenau, D.O., President of ACEP and we will all work collaboratively to insure the continued growth and success of emergency medicine. Unlike what the culture may have been in the past, ACEP and ACOEP have a very strong and collegial relationship that is focused on ensuring support for the entire “House of Emergency Medicine.” The current focus of ACOEP is to insure that as the environment changes we must evolve as well. We will continue to place a major emphasis on our student members to insure that you have every opportunity possible to obtain training in Emergency Medicine. We have more student members than Osteopathic Emergency Medicine GME positions; therefore, we know that many of you will have to obtain training in Allopathic programs. As we work to insure your future, we want you to recognize that and maintain your allegiance to ACOEP in the future. We continue to grow, but also know that we have not historically been able to maintain an ongoing relationship with those who don’t do Osteopathic GME training. I wish that I could provide you with all the answers as well as a roadmap on what the future will look like. However, as I write this letter to you the 2014 AOA House of Delegates is two weeks away and the final vote on the Single Common Pathway for GME will take place at that time. Please know that we understand your confusion over this issue, but rest assured the ACOEP is here to support you and make sure that you are taken care of to the best of our ability. +

page 13


Therapeutic Hypothermia and Targeted Temperature Management

The Fast Track

By Alexandra Murray, OMS-IV, OU-HCOM Spring 2014

T

he concept of cooling patients after a traumatic injury has been documented throughout history as far back as Hippocrates and the ancient Egyptians 1,2. In the 1930’s and 1940’s the concept of therapeutic hypothermia took form after there were several reported cases of hypothermic drowning victims that were successfully resuscitated. However, it wasn’t until the 1950’s that research studies were designed to investigate the role of therapeutic hypothermia in treating patients with traumatic brain injury and cardiopulmonary resuscitation.2 The initial research studies were uncontrolled and often used deep hypothermia (usually <30°c) for variable periods of time.2 Based on this preliminary research there were some positive outcomes associated with hypothermia, but the results were so unpredictable that the treatments were abandoned.2 In the 1980’s therapeutic hypothermia re-emerged as a potential treatment after animal studies showed neurological benefit from mild hypothermia (32°-34°c). 2 In addition, with the evolvement of intensive care units, the quality of care and side effects of hypothermic treatment became much easier to manage. 2 In 2002, there were two landmark randomized control trials that investigated the role of mild hypothermia in the neurologic outcome and overall mortality of patient’s following out-of-hospital cardiac arrest.3,4,5 The Hypothermia After Cardiac Arrest (HACA) Study Group concluded that therapeutic mild hypothermia for 24 hours increased the rate of favorable neurologic outcome and reduced mortality in patients successfully resuscitated after cardiac arrest due to ventricular fibrillation.4 The Bernard et al study group also concluded that treatment with moderate hypothermia for 12 hours appeared to improve outcomes in patients with coma after resuscitation from out-of-hospital cardiac arrest.5 Based on these studies and other research, therapeutic hypothermia protocol was incorporated into the 2003 advisory statements by the International Liaison Committee and European Resuscitation Council and the 2005 American Heart Association Guidelines for ACLS and Emergency Cardiovascular Care.6,7,8 During the past decade, the topic of therapeutic hypothermia has continued to gain international attention. Further research has demonstrated a continuous benefit from using therapeutic hypothermia and a 2011 meta-analysis of randomized controlled trials found that conventional cooling methods improves both survival and neurologic outcomes at hospital discharge for patients who experienced cardiac arrest.9,10 However, a randomized control trial published by Nielsen et al in December 2013 evaluating a target temperature of 33° versus 36° after cardiac arrest reported some conflicting results. Nielsen concluded that in unconscious survivors of out-of-hospital cardiac arrest of presumed cardiac cause, hypothermia at a targeted temperature of 33°C did not confer a benefit as compared with a targeted temperature of 36°C.11 Nielsen’s research has stimulated debate about whether cooling patient’s to 33°C is necessary in post-cardiac arrest patients or if avoiding fever and natural stress responses

page 14

with normothermia should be the primary goal of targeted temperature management. In addition, it can be argued that cooling and maintaining a patient at 36°C in the ED and ICU is much easier to manage compared to keeping a patient at 33°C. Many proponents of therapeutic hypothermia argue that Nielsen’s conclusions cannot be generalized to the average population and should not influence post cardiac arrest guidelines. Nielsen and colleagues have responded to backlash from their article by reporting that they do not advocate abandoning any temperature management on the basis of the results of their trial but more research is needed to clarify how to best adjust temperature or other interventions to each patient’s illness.12 The take home message based on the most recent literature is that modern, aggressive care that includes attention to temperature is effective and improves overall survival when a patient is hospitalized after CPR.13 Furthermore, hyperthermia should be avoided in the post-cardiac arrest patient.13,14 Additional research is needed to determine why targeted temperature management is effective at improving neurologic outcomes so that it is possible to refine protocols and define subgroups that would benefit from individual therapies. While there are dozens of proposed mechanisms for the underlying protective effects of hypothermia, there is still no unified understanding of how this treatment works.2,12 With that said, targeted temperature management and improvements in resuscitation have made a dramatic impact on overall patient morbidity and mortality. In contrast to a decade ago, one half instead of one third of patients with return of spontaneous circulation after CPR can expect to survive hospitalization with improved neurological outcomes.13 Hopefully, with further research and targeted temperature management we will continue to see improvement in emergency resuscitation. +


It’s never to early to start thinking about the Fall Conference in Las Vegas, NV, October 111h-13th, 2014. Visit www.acoep.org/fall for more details about pricing and how to register.

Spring 2014

The Fast Track

Are you into POLITICS? Want to make sure you are in the know about issues that face you as an osteopathic physician? Then check out the Grassroot Osteopathic Advocacy Link, by visiting www.osteopathic.org and search GOAL

Resident Chapter Highlights SATURDAY 10/11 9am - Jeopardy (by Rosh Review)* 11am - Dental Lab 1pm - Membership Meeting and Elections* SUNDAY 10/12 10am - Residency Fair* MONDAY 10/13 10am - Airway Shoot Out CHIEFS COLLEGE - Monday 10/13 * denotes required attendance for travel reimbursement page 15


The Fast Track

Spring 2014

WILDERNESS MEDICINE Special Edition

page 16


A Skill Set For Everyone

Spring 2014

The Fast Track

By Veronica Coopersmith, DO FAWM Candidate

”Oh

yea, wilderness medicine. Like snake bites and plant medicine, right?” That’s often the response I get when I tell people I would like to pursue a career in wilderness

medicine. Honestly, that wasn’t too far off from my initial idea of what this “wilderness medicine” field entailed. It sounded like it would bring me to epic destinations with cool outdoorsy people getting paid to do the awesome expeditions I have dreamed of my whole life. But the further involved I got, the more I realized just how useful wilderness medicine is for healthcare providers in all fields and any lover of the outdoors. So let’s think through a few scenarios scratching the surface of how useful wilderness medicine can be, beyond treating poison ivy. Scenario 1. You are the expedition doctor taking a group on a high altitude trek when one of the team members gets a sudden chest pain and severe shortness of breath. You percuss his chest and hear the hyper-resonant drum sounds of percussing a spontaneous pneumothorax. So you know what to do: decompress it with a large-bore needle, put in a chest tube, get a chest X-ray and an EKG.... Except you’re in the woods with just a trusty 12 gauge needle which you carry in your medical pack. Having attended many wilderness medicine conferences, you decompress his chest, make a slit in the 5th intercostal space over in the mid-axillary line, shove one end of a Camelback tubing in the pleural space and put the other end in a water bottle to make your modified chest tube to water seal. You call a chopper, evacuate the patient, and save a life in the woods.

page 17


The Fast Track Scenario 2. You are hiking in Rocky Mountain National Park with your family when your child Spring 2014

starts saying his head hurts, he’s nauseated, then he starts acting “weirder” than his already weird self. Your wife says he’s dehydrated, gives him water and tells him to stop being silly and to keep hiking. Having attended a wilderness medicine lecture on the dangers of high altitude, you realize this might be HACE (High Altitude Cerebral Edema). You make a modified kid carrier for him out of your backpack, put the tike on your back, and get him to lower altitude as quickly as possible. He starts feeling better and acting like himself once you go down a few thousand vertical feet. You bring him to the hospital to get checked out, and realize you might have saved your kid’s life from a deadly danger of high altitude. Scenario 3. You are on a multi-day trek with friends when you come across another trekker who fell and might have broken his leg. You learned at a wilderness medicine conference how to diagnose long bone fractures without an X-ray, so using your stethoscope and a tuning fork to listen to the bony vibrations, you decide that it is likely an isolated femur fracture. You use more wild med skills to rig a traction splint out of some paracord, sticks, duct tape, and a camping mug, package up the patient into a homemade litter carry, and safely get them to a landing zone for a helicopter. Another life saved in the backwoods.

Being an emergency medicine doctor obviously helps me be a better wilderness medicine doctor... However, wilderness medicine also makes me a better emergency medicine doctor.

All three varied scenarios have one crucial thread in common: just one person who knew basic principles of wilderness medicine was able to save a life. A little bit of knowledge can go a long way when you are the only hope someone has for hundreds of miles. One tid-bit of information taken from a single lecture or conference can make the difference between life and death for a stranger you happen upon in the woods, a team member or loved one, or even yourself. Being an emergency medicine doctor obviously helps me be a better wilderness medicine doctor: traumas, ensuring the ABCs, analgesia, orthopedic problems, etc. The translation from ER to wild med is obvious. However, wilderness medicine also makes me a better emergency medicine doctor (or a better primary care provider in rural environments) and that translation isn’t so obvious. To page 18


The Fast Track

tto understand how to practice medicine with all of our ccrutches and safety nets removed. You need to figure

Spring 2014

understand how to provide medical care in the wild is u

out what’s wrong without the fancy labs, x-rays or CT o sscans, and without the lifelines of colleagues, consults, tthe interwebs, or smartphone apps. It’s just you and tthe patient: REAL medicine. Then, when you figure out what’s wrong, you need to fix it, but without commercial chest tube trays, intubation kits, or ventilators with settings. Instead, you actually need to understand the physiology and pathophysiology of presenting problem and understand it enough to fix it with camelback tubing, a pen, duct tape and perhaps some stray pieces of clothing. Even though I may never need to MacGyver a chest tube in the wilderness, the fact that I know how to helps me really understand what’s going on in my patients’ chest with a pneumothorax. And even if I never have to treat hyperkalemia from a crush injury, I am so much more aware of the presenting signs and symptoms out of the fear of having to diagnose it in the woods without an EKG or BMP. It’s back to the basics before everything was protocolized. Just you and your brain out there, remove all the red tape of insurance companies, charting and covering your butt by getting a CT “just in case” it’s a small subdural giving the headache. This is the part of medicine that I (and many of us) got into this business for...NOT plugging the answers to the Centor criteria questions into MD Calc and giving antibiotics because of the number it spits back to me. It removes everything between you and the patient. Wilderness medicine is a wonderful ul mix of humbly going back to the basics of an old History and Physical textbook, with using some common sense and a little bit of crafty ingenuity. Ultimately, the real definition of wilderness medicine as I’ve learned through countless opportunities is that it applies in every scenario that you are alone with limited resources, in a harsh environment, with the odds against you. So whether it be in the mountains, your family vacation, a local disaster, a third world country, or a rural clinic, the application of wilderness medicine is absolutely limitless, a skill set to be harnessed by all.+

page 19


The e Fast Track

Spring 2014

HISTORY OF PRESENT ILLNESS: Patient X is a 32 year old Caucasian male who presents to the emergency department with his right hand tightly wrapped in an ace bandage and a dead snake suspended on a stick in the left. Earlier in the afternoon Patient X was walking on the trail near his property and came in contact with a snake. He attempted to kill the snake with a stick lying nearby. During the struggle Patient X reached down to pick up the snake thinking that it was dead. He was then bitten by the snake on the dorsal aspect of his right hand. Patient X finished killing the snake, wrapped his hand, and drove to the emergency department. In the 45 minutes it took to drive to the ED Patient X developed facial swelling, dizziness, and shortness of breath. The snake was identified as a young rattlesnake. The patient was immediately placed on an O2 monitor and an IV was started. PHYSICAL EXAM: Vital Signs: BP 69/42, HR 111, RR 26, SpO2 94% on room air. Temp is 98.0. General: Mild respiratory distress, no audible wheeze or stridor, speaking in full sentences. Diaphoretic. Eyes: PERRL with bilateral eyelid swelling, EOMI Cardiovascular: Tachycardia, no R/M/G Lungs: CTA, no wheezing or stridor Extremities: Erythema and edema on dorsum R hand to mid-forearm Labs and Testing: EKG: Sinus tachycardia with nonspecific ST depression. Labs: WBC 24.4, HgB 21.0, HCT 62.7, PLT 105 NA+ 138, K+ 3.9, Cl- 108, BUN 14, Cr 1.4, CO2 14 Troponin I 0.139 and 1.48 (3 hours later)

page 20

Total CK 286

CKMB 4.0 PT >100, PTT >300, Degradation Products >40

Fibrin

DISCUSSION: Patient X is a prime example of the “typical” snakebite victim; 70% of victims are males over the age of twenty7. With around 45,000 cases of snake bites reported each year in the US and an average of incidence of 3-4 bites per 100,000 people1, 2 , knowing how to care for such exposures is extremely important in emergency medicine. Most snakebites result from nonvenomous snakes: however; it may be extremely difficult to tell if the snake is venomous based on the initial symptoms. Venomous snakes in the United States are broken up into two categories: Crotalinae, which consist of copperheads, rattlesnakes and water moccasins and Elapids, which include coral snakes, cobras and mambas (only coral snakes are seen natively in the United States)3,6. 99% of snake bites are Crotalinae. Crotalids produce a hemotoxic venom, which is not usually fatal unless injected into the vein. Effects from these bites can include a variety of effects such as local tissue spread, compartment syndrome, nausea, vomiting, chills, diaphoresis, rhabdomyolysis and coagulopathies and rarely neurotoxicity (seen commonly with Mojave rattlesnakes)3,4 . A potential and significant cluster of hematological abnormalities can

occur with Crolatid bites in a syndrome called “defibrination” syndrome. These abnormalities resemble disseminated intravascular coagulopathy (PT/PTT elevations, low platelets and elevated fibrin degradation products) but occur via a different mechanism1. Capillary permeability changes leading to leaking of albumin, electrolytes and RBC’s. An initial elevation in hematocrit may also be seen due to edema and hemoconcentration. These values then decrease as a result of blood loss and intravenous fluids. Mojave rattlesnakes bites are unique in that they present differently than most Crolatid bites. Many texts compare their bite to that of Elapids. They produce a neurotoxic effect, which can quickly lead to respiratory depression and neuromuscular weakness. It is rare that Mojave bite produce severe local reactions. It is recommended that antivenin be started immediately after exposure or as soon as symptoms develop3,6.Patients bitten by Mojaves should be monitored for 12-24 hours, with a low threshold for intubation


Snake Bites

Spring 2014

The Fast Track

A case study and discussion Nate Moore, OMS-III @ VCOM and respiratory support.

Pressure immobilization (wrapping the wound with an elastic bandage to reduce lymph spread and systemic absorption) is NOT advised for Crotalinae snakes but may be useful for Elapids1,4,.

Effects of Elapid bites are usually more systemic, rarely cause local reactions but do cause muscle weakness, dysphagia, hypersalvation, respiratory depression, and respiratory arrest1,4 . Effects of Elapid bites occur from toxins that work by blocking acetylcholine receptors at the neuromuscular junction.This can lead to inhibition of cardiac and skeletal muscle.6

Do Immobilize the patient as much as possible.

The goal of therapy is to rapidly treat with antivenin.

Do cleanse the wound with water.

Exotic snakes are becoming more popular in United States due to ease of international travel and access to the internet. Private snake owners and zoo’s are often have identification cards and snake specific antivenins onsite. However many snakes are not legally registered, making photographic identification essential in treating their bites6.

Do loosely wrap and immobilize the affected extremity.

Do remove anything that causes constriction-rings, watches, jewelry or tight fitting clothing.

Do photograph the snake if possible. Do transport the victim to an emergency department as quickly as possible. In the ED1,3,4:

IN THE FIELD- DO’S AND DON’T’S3,4: Do not use a tourniquet. Do not attempt to use suction or commercially sold “snake suction kits”. Do not attempt to capture the snake.

Assume all snake bites are venomous until proven otherwise. Dry bites may occur is as many as 20-50% of Crolatids. Patients with asymptomatic snake bites should be monitored for at least 12 hours for status changes5. The first step, unless you’re experienced with venomous snakebites is to call the poison control center for further advice.

page 21


The e Fast Track Antivenin is usually given based on the level of severity of the bite. Spring 2014

Snake bites are broken up into a four part grading scale6: Grade 0: no envenomation suspected- Minimal pain and less than 1 inch of local spread in 12 hours. Grade 1: Minimal- Bite marks present. Moderate pain at bite site and local spread is one to five inches. No systemic involvement after 12 hours Grade 2: Moderate- Moderate pain, not localized. Pronounced edema moving toward trunk, with petechiae. Fever, nausea and vomiting are usually present. Grade 3 :Severe- Worse local involvement plus systemic involvement: elevated WBCs, D-dimer elevation, hematuria, myoglobinemia and low platelets. Also CK, PTT, PT elevations. Grade 4: Severe-The same as Grade 3 occurring more rapidly and with worse local tissue destruction possible bleb formation and necrosis. These bites may rapidly progress to muscle fasciculation’s, coma and cardiovascular collapse.

if anaphylaxis occurs). Obtain Vital signs and place the patient on a cardiac monitor. Begin serial wound monitoring (measuring local reaction with marker). Check for initial labs including CBC, BMP, CK, PT/PTT/INR, fibrinogen, D-Dimer, UA and EKG. A UA should be performed an hour after the bite to assess for hematuria,myoglobinuria and renal dysfunction. Antivenin is used for patient’s classified suffering moderate to severe reactions. These patients include those with changes extending beyond bite, abnormal coagulation or any systemic symptoms. Antivenin for Crolatids causes hypersensitivity, serum sickness and possibly anaphylaxis in up to 16% of patients. It works best if administered in the first 6 hours but can be used for coagulopathy problems as late as 24 hours after envenomation. Management for elapid bites:

Management for Crolatids3:

Perform serial neurological and respiratory exams.

Place two peripheral IV’s (access of the second site may be needed

Intubation may be required if respiratory distress is present.

Give antivenin as soon as possible if available. Some experts recommend giving antivenin immediately regardless of symptomatology. Elapid antivenin is no longer produced in the United States but may be available in states with native coral snakes. Hospital admission is most likely required4. Assessment and Plan: Patient X is suffering from a Grade 3 (severe) reaction. He began to show symptoms very quickly after the bite and they rapidly worsened. Poison control was consulted. After Patient X’s initial labs he was given a total of 5 vials of antivenin during his time in the ED. Repeat labs were scheduled for every four hours. His hand was marked every 15 minutes, to monitor local progression. He was given a fluid bolus. His coagulopathy worsened initially and his platelets decreased. His hemoglobin and hematocrit although originally elevated, dropped profoundly after treatment. Both effects were likely caused by cell lysis from the venom. His blood pressure did not respond to fluid resuscitation and he was given IV Dobutamine. He was admitted to the ICU to continue treatment with antivenin. Hospital Course: Patient X was given a total of 19 vials of antivenin. Patient X developed acute kidney injury from the snake bite which corrected with hydration. Troponin levels were trended and eventually decreased. Repeat EKG was normal. Echo showed no structural damage. His blood pressure normalized. He had two episodes of hematemesis during his first inpatient day. All of the above completely resolved prior to discharge. The local bite site became progressively inflamed and was treated with IV antibiotics. He was discharged on PO clindamycin. The severity of the bite was most likely due to the fact that he was bitten by an immature snake who released an exaggerated amount of venom. +

page 22


Spring 2014

The Fast Track

Lisa Hrutkay D.O., FACOEP, FACEP In July 2010, I visited the National Boy Scout Jamboree at Fort A.P. Hill in Virginia. My youngest son was participating in the Jamboree that year with his troop. While touring the site and visiting the exhibits and activities there, my husband and I learned of the plan to move the Jamboree to a permanent site. The National Boy Scout Jamboree is an event held every four years and attracts thousands of scouts between the ages of 12 and 17. It has been held in many locations, but in the last several years has been held at the military base at Fort A.P. Hill located near Fredericksburg, VA. A few years ago, the U.S. military base stated that it would no longer be able to host the event and that a new venue was needed. Twenty-seven states applied for the opportunity to become the new home for the Jamboree and their proposals were reviewed by the Boy Scouts of America (BSA). They were judged in regard to the terrain and the types of activities to be offered; their accessibility via highway, air and railway systems; the support systems available in the area; and the proximity to the majority of the country’s population base. We were delighted with the announcement that the newly selected site was located near the New River Gorge in our home state of West Virginia! The 10,600 acre site was donated to the BSA by the Bechtel Corporation and has been named the Summit Bechtel Reserve. My husband and I have been involved with our local Boy Scout troop for 13 years with our sons and believe in the mission and values of scouting. We also have a great deal of pride in the beauty and resources available in our state. We are both medical professionals (my husband is a paramedic/firefighter and I am an emergency physician) and we wanted to become involved in the planning for the medical support for the Jamboree. I was placed in contact with Steven Eshenaur, D.O., Summit Medical Director, and initiated a discussion for a novel idea. I proposed that we hold a wilderness medicine rotation for medical residents and students in conjunction with the Jamboree. This would be an excellent educational experience for the participants, as well as providing additional manpower for the Summit Medical Staff. To our knowledge, this had not been done before with a Jamboree event. In fact, there had never been a wilderness medicine elective ever offered through an osteopathic training program in the U.S! This would be a truly unique offering, and require a substantial amount of time and energy to initiate. I have been in practice as a board-certified emergency physician for 16 years and, admittedly, have had many of the early signs of burnout. For the last few years, the greatest satisfaction I have had in my practice has been the mentoring and teaching of residents. I also have a great love of the outdoors, especially outdoor activities in West Virginia. I began to view this project as method of tying together my love for training the residents, an exciting outdoor experience, and service to the scouting organization. It would also be an interesting and unique offering for the EM residency program at Ohio Valley Medical Center, where I serve as core faculty. I was given full support by our EM residency director, Joe Dougherty, D.O., and I set to work. Many challenges greeted me in the startup of this program. Development of the curriculum was actually the easy part. Even though no template existed in osteopathy, there are a few allopathic page 23


The Fast Track

Spring 2014

electives which did serve as a pattern for the curriculum development. I made contact with the Wilderness Medical Society for additional tio ona nall information info in form rmat atio ion n about abou ab outt ut util utilizing iliz izin ing g th thei their eirr lilist st o off wi wild wilderness lder erne ness ss medicine core material. They were e e helpful in this regard and it led to the elective having their endorsement and d accreditation for 24 hours toward d wilderness medicine fellowship. Some e of the other aspects of the development nt of the program were not as easy. We had d difficulty in locating free housing for the e participants. The West Virginia School ol of Osteopathic Medicine lent assistance e and was able to place some of the participants icipants in student housing available in the area and rented a house in the area for the remaining students and residents. One of WVSOM’s clinical rotation sites, Raleigh General Hospital in Beckley, WV generously offered a conference room with computer use and projection, and even fed us all in the cafeteria free of charge on the days that we were scheduled to be there. Then there was the recruitment of faculty for the program. The schedule was designed with two weeks of lectures and workshops on various wilderness medicine topics, then ten days on the BSA Summit delivering care to scouts and staff, followed by a final half-day outdoor scenario to utilize some of the skills learned. The Summit is located four hours from my home and my knowledge of local resources, at the time, was limited. I solicited members of the Jamboree medical staff who were already registered and received a great response from many physicians from all over the country who were coming to the event and were willing to lecture. Most of them, however, were not arriving in WV until a few days before the Jamboree. I was able to schedule their lecture times during the days just before the start up or actually on site at the Jamboree as a morning lecture. The remaining lecturers were some of my partners with Emergency Medicine Physicians (EMP), other local physicians with an interest in WM, friends of friends, etc. It was definitely challenging. I had great support from the local EMS providers, who helped out with supplying manikins and equipment for training. Also, the WV AirEvac Lifeteam personnel were especially helpful in assisting me with set up of workshops in basic rope rescue, swift water rescue, splinting improvisation and spinal immobilization, landing zone set up and lecturing on flight physiology. The recruiting for the Th participants in the program pa was wa limited, due mainly to the brief time prior to the start of the program. Dr. Dr Joe Dougherty sent the information out to the other osteopathic medical oth directors and the emails dir began to come in. The final be group of nine participants gro was wa made up of three EM residents, three FP residents, res

page 24


and three medical students. They were from Wheeling and Beckley WV, Ashland KY, and Louisville KY. With the time commitment and stress involved, along with my clinical practice, faculty responsibilities, and my second job as an aeromedical director; I questioned whether I would ever attempt such an undertaking again.

Spring 2014

The Fast Track

This commitment of time and energy did not go unrewarded. The rotation was very enjoyable and successful! All of the material presented was very interesting and the workshops were exciting. The time spent on the Summit was very rewarding also. They were assigned in groups of three’s to the areas of the Summit where the high risk for injury was predicted; the skateboard park, the BMX bike trails, and the mountain bike trails. They also spent one or two days with the EMS response units on site. The Jamboree attendance for 2013 was 34,000 scouts and 3,500 staff, thus some lecture time involved planning for mass disaster response. Thankfully, no disaster occurred and the majority of the problems seen were heat-related illnesses and orthopedic injuries. The participants had the opportunity to provide care at the scene of the incident, which was a new learning experience for them. They integrated well into the medical staff there and their presence and assistance was appreciated. After the Jamboree, the final scenario exercise served as the final wrap-up of the program and challenged their new knowledge and skills. It involved a simulated ATV accident with two moulaged victims lying at the bottom of a 35’ cliff in a creek bed. The participants had to set up ropes for rappelling and hauling the victims, rappel down to access them and assess and treat their injuries. They then had to immobilize them, secure them in stokes baskets and evacuate them to the top of the cliff. The victims were then transported to an area that they set up as a landing zone and they were then evacuated by medical helicopter.

The residents enthusiasm and group. I t and d students t d t had h d tremendous t d th i d were a very attentive tt could not have asked for a better group of participants for this rotation. They made this a very rewarding experience for me also, and gave my entire career new meaning. This has definitely been the most rewarding accomplishment of my EM career. I owe so much to so many individuals and groups who assisted me with this project. My greatest thanks is to my partners at EMP of Ohio County in Wheeling, WV who supported my ambition and worked extra shifts for the entire month of July to allow me to take this “sabbatical” month downstate to run the rotation. If I can offer any advice from what I have learned from this experience, it is to find your passion and follow it. It may reinvigorate your career! Oh, and yes, we will do this again in 2014!+

page 25


The Fast Track

Spring 2014

A

fter two years stuck behind desks covered with books, fingers cramped from notetaking and highlighting, I am overjoyed to enter the clinical world. Medical school has demanded a lot of sacrifices. One of the hardest sacrifices has been enjoying the view of the Appalachian Mountains outside my window instead of hiking its trails. Unfortunately, finding mentors who combine their need for the outdoors with the art of medicine has not been easy. Yet, in planning the journey ahead, I am hopeful to find kindred spirits living a life I could emulate. Rotations are a chance to experience a seasoned physician’s career and see if you can imagine yourself following a similar path. Of all of the incredible hospitals and mentors to work with, nothing can combine a student’s passion for medicine and the outdoors like a wilderness medicine rotation.

A Rotation on the Wild Side

Nestled in more than 10,000 acres of beautiful forest and next to the New River Gorge National Park, known for its challenging whitewater rafting and world-class rock climbing, sits the Summit Bechtel Reserve. Over the past few years, Dr. Lisa Hrutkay and several others worked to start the first osteopathic wilderness medicine rotation in this beautiful location in southcentral West Virginia. The rotation, now in its second year, is co-sponsored by the Ohio Valley Medical Center Emergency Medicine Residency and the West Virginia School of Osteopathic Medicine. Open every summer from mid-July through August, fourthyear medical students and residents are invited to participate in a two- or four- week experience. The Summit reserve provides a three-season high adventure experience for Boy Scouts from across the nation. Every four years, this location hosts the National Boy Scouts Jamboree, bringing more than 30,000 participants and 300,000 visitors. During these dates, students rotate for four weeks and during the off years, the rotation is two weeks long. Sasha Rihter, OMS-III at WVSOM

page 26

Besides working with medical staff onsite at the Summit, students participate in workshops and lectures. Outdoor workshops take advantage of the austere environment and simulate real-world situations as much as possible. Rope rescues take place on nearby rock walls and students wade into the white waters of the New River for simulated swift water rescue. Working alongside Ohio Valley’s medical helicopter teams, students can practice preparing aeromedic pick-ups and search and rescue scenarios. Stepping indoors, students learn about a wide variety of topics including


altitude sickness, hypothermia and heat-related emergencies, decompression sickness and diving emergencies, wilderness orthopedic injuries, disaster planning and more. As a past participant explains, students should “…expect to switch gears quickly; one minute you may be relaxed and enjoying your surroundings, the next you may be faced with making serious medical decisions quickly.”

Spring 2014

The Fast Track

“The most beneficial aspect of studying and practicing wilderness medicine is the practicality and utility of it for any physician, especially those who love the outdoors,” Dr. Michael Antolini noted, a second-year resident and former participant. During his experience, Michael felt the rotation, “showed me there’s more to medicine than clinic or hospital work, and there’s just as much opportunity to heal others and occasionally save lives outside the realm of the day to day grind in an office.” For some it was an eyeopening experience and created appreciation for those who deliver quality prehospital care: “It made me realize it can be a tough, dirty job, sometimes even dangerous. These guys do the job every day. It takes a special kind of fortitude to function like that under such high pressure and they deserve credit for that.” Before taking on such a challenge, successful participants suggest “mastering understanding of the body’s basic physiology” because it will help students “reason even faster when put in the unique and stressful situations” seen in outdoor emergencies. “These are critical decisions that shouldn’t be second guessed. Knowing this stuff could truly be life or death for someone,” Michael explained. He also suggests physical preparation because “you may find yourself actively assisting in evacuating a patient, or simply simulating this evacuation. Either way, all the knowledge you carry in your head doesn’t mean much if you can’t help your team carry you or your patient to safety if the situation demands it.” Somewhere along the next few years, we are supposed to develop an idea what our careers should look like. For so many of us still defining these parameters, trying experiences outside of the normal curriculum can help focus our goals and show us the type of physicians we should strive to become. For Michael, this rotation was an enlightening experience: “I belong outdoors. I love practicing medicine. (Wilderness medicine) allows me to do what I love where I belong. Isn’t that what we all strive for? Spending time on this rotation made me see that was possible.”

For more information on this rotation and contact information, please visit www.wvwildmed.com. +

page 27


The Fast Track

Spring 2014

Facing the Elements: An Interview about Wilderness Medicine By Julie Littwin, OMSIV @ CCOM

While Wilderness Medicine is a relatively new specialty branch of emergency medicine, the concepts of resourcefulness, creativity and mastery are not at all new to medicine. As Dr. Eric Lombardi experienced as a 4th year medical student rotating through University of Massachusetts’s elective, training involves learning ďŹ rsthand the wilderness survival skills to take care of yourself such as basic avalanche awareness and drinking water management. Medical training involves treating such conditions as hypothermia, dressing wounds, splinting orthopedic injuries as well as working with as a team without the conveniences found in the hospital setting. page 28

Recently, Dr. Lombardi sat down with me in order to share his experience with The Fast Track.


The Fast Track

My rotation took place over three weeks in May of 2013, 10 days after the Boston Marathon bombing. The reason I mention that is some of the same doctors who responded and treated victims of the Boston Marathon bombing were our instructors for the trip. It made the concept of emergency preparedness all the more relevant.

Spring 2014

When did the Wilderness Medicine rotation take place?

How was the rotation structured? There were three main training areas: alpine, water and dessert evolutions in Utah on Mt. Pfeifferhorn in Red Pine Canyon, along the Green River, and in the Needles section of Canyonland National Park. We had a two day training, learned how to properly pack our bags then formed teams. There were 12 medical students from various schools around the country. I was the only D.O. student. There were also eight residents, two Wilderness Medicine Fellows and ER Physicians who taught us and guided us along the way. What was the experience level of the students on your month? There was a wide variety of experience levels in the group I was with. One student had never hiked before while others were in the Marines and the Army Rangers before medical school. The instructors trained all of us well enough to make it through. What were the goals of the rotation? The rotation was as much about wilderness medicine as it was about how to organize and work with a team through mock scenarios as well as treating hikers as well as some of our own team along the way. We learned to treat common injuries and conditions while also learning how to manage a mass casualty event per FEMA standards. Lectures were ongoing and each medical student gave a lecture on a previously prepared topic. The focus was on doing the best medicine you can as a team while not having access to everything you normally would in a tertiary care center. For example, how do you check for anemia without a CBC? You look for conjunctiva pallor, dry mucous membranes and tachycardia. How were the mock scenarios incorporated? Only the students ran the scenarios which forced us to step up. Those are the types of situations you really learn in. The residents, fellows and physicians were the actors. They really got into their roles and it made it feel very real. We even had to restrain one of them when reenacting a boat accident rescue when they were rowdy, cursing and ďŹ ghting. They got into it! We never knew when the mock scenarios were going to happen or what the scenarios would be; whether at the end of a 12 hour day of river rafting or page 29


The Fast Track

Spring 2014

at 3 in the morning after finishing a hike, we just had to be ready. So we organized beforehand by assigning roles of leader, safety officer, communicator, etc. so when a situation popped up, our roles were already known. We could give each other tips, such as, “Maybe we should check his glucose,” but everyone had their assigned job. Can you tell us about one of the scenarios you participated in? In the alpine section, an avalanche was simulated then we had to find and rescue the Resident that volunteered to be the trapped skier. We found him by taking long sticks and poking them down into the snow until we felt a change in density, which was his body. It took us 9 minutes. If he did not have the oxygen with him for the training, he may not have survived, since most victims in an avalanche die within 5-7 minutes due to hypoxia. The trapped said the scariest part was hearing the creaking of the snow above him from us walking and being jabbed by the poles; he couldn’t hear us talking since no sound is transmitted but he did think it was creepy to hear the ice and snow creaking between us and him.

Residency Spotlight: Genesys EM Program, Grand Blanc, MI

How was your performance assessed after the scenarios were completed?

We all got feedback from the Doctors leading the trip and they did not hold back. If a scenario was not going well, they would call it: in real life, the victim may have died and they try to keep it as close to real life as possible. For that, you need to have tough skin. Was there ever an instance when you had to work around acts of nature?

Yes. While on the Green River, I was the leader of a scenario where we were evacuating a mountain climber who fell and broke his femur. Our goal was to stabilize him and safely transport him back to the river for a water evacuation, when the safety officer noticed a cougar was stalking us. We had to abandon our plan. That night we all slept close together instead of setting up tent with lots of space and privacy in between! Another night I chose not to sleep in my own tent after hearing a rattle from a rattlesnake coming from inside. What challenges did the desert present? Packing in water- we packed in 12 gallons of water and had to ration it for ourselves over 5 days. We also didn’t bring tents so as to lighten our already 70 pound packs so we slept in the open air. One night it rained as we were going to sleep. And what happens when the dessert sees a couple minutes of rain? Wildlife comes out. All of a sudden spiders as big as my palm were coming up out of the ground and crawling on us. No one slept much that night since we were busy fighting off spiders. I don’t usually mind spiders, but these were huge and there were so many of them. Did any real-life emergencies need treatment during those three weeks of training?

page 30


For sure, a couple of times. We treated a scorpion sting with wound care, giving opiates for pain and hydrating our teammate. There were two broken noses and a couple lacerations. Thankfully, we had all the gear to fix them in our first aid kits. What better timing to get a laceration than amongst a bunch of ER Physicians! The resident who volunteered to get stuck in the avalanche was treated for minor hypothermia. We also treated many hikers for problems such as blisters, sprained ankles and dehydration. The first couple days of the alpine trip, we all experienced altitude sickness. Thank goodness no one’s symptoms were so severe that they needed treatment or to descend and our symptoms were self-limited.

Spring 2014

The Fast Track

Can you share some of the logistics to getting involved? There are multiple programs and scholarships available for Wilderness Medicine rotations. I had a great experience with University of Massachusetts. I contacted them and was put on a waiting list until after the match. Once I successfully matched into ER, they got back in touch with me to finish setting everything up for the three week rotation in the spring. They provide a very detailed list of gear you need to bring with you such as a sleeping bag for below 0 temperatures as well as list of what gear can be rented on arrival to Utah that you would not invest in, such as snowshoes. How do you feel those three weeks in the wilderness has helped you in your training this first year as an intern? The Wilderness Medicine Rotation was the absolute best experience to prepare me for intern year as an ER Resident. I may not ever find myself in an avalanche again but I will definitely take care of more patients with hypothermia in Chicago. My first week as an intern, my patient, a surgical patient that was transferred to the ICU and crashed. The surgeon was back in a case and there were no other ACLS trained residents in the immediate area. Everyone was looking at me, and I acted. I know it was the training I got with the UMass program that helped me to take responsibility when things start to go wrong, as in that moment. + -Dr. Lombardi is wrapping up his intern year as an Emergency Medicine Resident in the St. James Hospital/ Midwestern University OPTI Program. He is expecting his first child with his wife, Cathy, in July. Links: Listing of Wilderness Medicine Opportunities for medical students: http://www.emra.org/committees-divisions/wilderness/wilderness-electives-for-medical-students/ UMass website: https://www.umassmed.edu/emed/rotation/wilderness-medicine-elective/index.aspx

page 31


The Fast Track

Spring 2014

Drowning by Ben Abo, DO @ Mount Sinai Medical For most of the US, summer is upon us. For us here in Florida, well… it is always summer. With warmer weather, comes more pool time, lake time, river and beach time. Of course, drownings happen in all types of water all year round but with certain seasons they become more prevalent. Drowning affects all ages, but is a leading cause of death ages 1-4 and according to the Florida Department of Health, “annually in Florida, enough children to fill three to four preschool classrooms drown and do not live to see their fifth birthday”. This is a heartbreaking statistic. I know last month in my emergency department, I worked over six drownings personally. Florida is a hot spot, but we are not alone. Globally over 380,000 drowning deaths occur per year and 7.6 million injuries, not including boating accidents, suicides or homicides and in the US over 4000 annually. In other words, the US has 1.2 drowning deaths per 100,000 deaths annually. Interestingly,

page 32

drowning

has been an issue for hundreds of years. The first ever recorded treatment for drowning was 1237 BC when someone fell into a river in ancient Egypt. In fact, there are hieroglyphics and ancient Chinese graphics that describe attempts to treat such events. Over time, like other fields of health and disease, we have made many strides in understanding the pathology and best treatment practices. It’s hard to think modern ventilator management came from things like tobacco enemas where smoke was literally blown up victims’ rear-ends in the 1700s to resuscitate victims. Before getting into the pathology and the important treatments of drowning, it really is important to understand definitions. Those that work with me may think it is just my OCD semantics, however my friend Andrew Schmidt DO MPH, one of the directors at Lifeguards Without Borders, says it well: it is so important to be speaking the same language. Just like when we are dealing with other diseases such as diabetes

or hypertension, dealing with multiple doctors, you want them all speaking the same language and all have the same understanding of what diabetes or hypertension is. There is a review article in Resuscitation from 2005 that looked through 60 years of literature and found 33 different definitions of “drowning”; different terms such as “wet drowning” and “near drowning” also were found and further confused things. This led to the 2002 World Congress on Drowning held in Amsterdam. One of their major goals was to come up with a universal, standard definition of drowning. What did they come up with? Drowning – The process of experiencing respiratory impairment due to submersion or immersion in a liquid. From that primary definition there can be one of three possible outcomes – injury, no injury or death. Furthermore, they recommend not using confusing terms such as near drowning, secondary drowning, etc. in order to simplify things. Throughout everything that happens with drownings, the important endpoint is our true enemy: hypoxia, or lack of oxygen. This is the primary cause of all systemic injury and death associated with


Spring 2014

The Fast Track

drowning. This makes sense, right? You get the airway under water, the victim attempts to resurface and voluntarily protects his or her airway (holding breath), but then there is an involuntary attempt to inhale oxygen. Next is one of two things: laryngospasm or no laryngospasm. After this comes respiratory failure, whether it is from obstruction or pleural effusion or alveolar damage… all of which lead to hypoxia. This is why the prehospital and hospital treatment is oxygen. I had the pleasure of hearing Lifeguards Without Borders speak and loved the following reminder: 21 > 0. 21% of ambient air that oxygen makes up is better than nothing. If you do not have an O2 tank to hook up yet, don’t wait! Furthermore, a lot of drowning victims can have quite a bit of foam coming from their airway. Suction some, but it will keep coming, and if you keep suctioning it they are getting no oxygen. So what do we do? Ventilate through it if possible! The more recent resuscitation guidelines for both bystanders and professional rescuers advocates prioritizing compressions and our ABCs have been replaced with CAB. This makes sense and has been teased out quite well. If I have a patient that the story fits, I treat it and think it a cardiac cause – especially if the starting rhythm is ventricular fibrillation or ventricular tachycardia. However, in most drownings, it is still important to stress oxygenation and ventilation. In fact, if you actually look at the minutia of the AHA guidelines and the modifications, they specifically state “CPR for drowning victims should use the traditional A-B-C approach in view of the hypoxic nature of the arrest”. Recently while I was on an MICU

rotation, a medicine intern made the statement, “Well how do we know ocean rescue didn’t do the Heimlich maneuver? It is a drowning”. My response – they didn’t and they shouldn’t. There has been plenty of forensic research showing there actually is very little water in lungs. Time spent getting this bit of water out is time not being spent oxygenating. No professional association suggests or recommends the use of the Heimlich maneuver. Now from here we could get into ventilator management and critical care as things progress, but honestly I think that would be a better topic to properly dissected out. What I think is more appropriate for us to address and think about is what do we do with these people when they come to my ED? Those in respiratory arrest or significant and obvious respiratory failure have an obvious answer. What to do with those that appear fine or almost fine is more of a gray area. Dr. Szpilman out of Brazil published a great study in Chest back in 1997. This looked at a huge

page 33


The Fast Track

Spring 2014

sample size of over 41,000 drowning patients. All patients were basically assessed and 93% of them were released on scene, leaving 2,304 that received hospital care. The important thing from this is that he established six subgroups based on mortality rates and clinical criteria. This helps us decide who needs a close eye for a bit versus those that can be let go. Based on this and other emergency medicine principals, patients who are asymptomatic on presentation to the emergency department, maintain a normal room air oxygen saturation, and have no chest radiograph or arterial blood gas abnormalities can be discharged safely after an observation period of about six hours. Even then, specific instructions and warning signs of when to return to the ED need to be given to the patient and any family or friends. Hopefully this little review will help prepare you for the upcoming “busier” drowning season, but do not think that things only happen “in seasons”. We are recognized as a field more for what we are prepared to do and not so much just what we do. Furthermore, there are a number of great resources and terrific groups doing amazing things. You can always count on quality water rescue and drowning related material from Lifeguards Without Borders and their directors Dr. Andrew Schmidt and Dr. Justin Sempsrott. I know that I sure have learned a lot from them over the years and have no doubt that in this journey of lifelong learning, I will continue to do so. With this I leave with the motto of the Carolina Wilderness EMS Externship: esse quam videri. +

Resident Chapter Highlightss SATURDAY 10/11 9am - Jeopardy (by Rosh Review)* 11am - Dental Lab 1pm - Membership Meeting and Elections* SUNDAY 10/12 10am - Residency Fair* MONDAY 10/13 10am - Airway Shoot Out CHIEFS COLLEGE - Monday 10/13 * denotes required attendance for travel reimbursement

page 34


Truly Life Saving Skills of EMS

Spring 2014

The Fast Track

Stephen J. Vetrano, DO, FACOEP, FACEP, EMT(I)

There are a plethora of clinical skills taught to EMT’s and paramedics, but there are only five things that are truly lifesaving in the field. The majority of what is done is done to accelerate the patient’s care through the emergency care system. What is remarkable about these truly lifesaving skills is they are, for the most part, at the Basic Life Support level. These life-saving skills are listed as follows, and will be explained in greater detail: 1. Airway Management; 2. Needle Decompression/Needle Thoracostomy; 3. Cardiac Defibrillation; 4. Tourniquet; and 5. Epinephrine Auto Injector. Airway Management is an all-encompassing skill, starting from the basics in CPR training, all the way through to physician specialties including emergency medicine and anesthesiology. It is the “A” of the ABC’s. No one dies without an attempt at airway management. Opening, clearing and maintaining an airway gets the patient breathing. This is truly life saving. How we go about doing it comes in several ways. First and foremost is positioning: the head-tilt/chinlift maneuver taught in CPR classes throughout the world. It is often taught airway management in the trauma patient needs to be done with care provided to protect the cervical spine; the jaw thrust maneuver does just that. However, recall the basic motions of the C-spine: the majority of flexion and extension, the very movement done in the head-tilt/chin-lift, occurs at the OAC1 joint; not the true cervical spine. Head-tilt/ chin-lift does have a role in the trauma patient, particularly in mass casualty situations where untrained or minimally trained providers may have a role, or when the jaw thrust is ineffective at opening and maintaining the airway.

Added to this would be airway adjuncts. These devices simply keep the tongue, the primary anatomic reason for obstruction, off of the airway. Both oropharyngeal airways and nasopharyngeal airways accomplish this. They are inserted in the position in which they lie: oral airways being measured from the corner of the mouth to the angle of the jaw, and nasal airways from the tip of the nose to the earlobe plus the diameter of the nostril, approximated to the size of the patient’s fifth finger. Care must be taken to lubricate the nasal airway, and the oral airway cannot be placed in patients with an intact gag reflex. Supraglottic airways go deeper. They sit above the glottic opening. One of the original such No one dies without an attempt at airway management.

device is the laryngeal mask airway. This device is best described as the mask of the bag-valve mask being shrunken down to fit over the larynx. Often used in controlled settings such as the OR, they also make it easier to intubate, either with specially made LMA’s or ingenuity. Combitubes are another type device, similar to its predecessor, the Pharyngeal-Tracheal Lumen airway (PTL); this device can be used as an endotracheal tube or as a supraglottic airway. It is blindly inserted and two pilot balloons are inflated: one inflates distally, the other proximally. Ventilations are then attempted in one of the two tubes of the combitube. The first tube is the pharyngeal tube. Air comes out of the end of this tube and cannot go back out, as the pharyngeal balloon seals the airway, and cannot go down the esophagus, as the distal balloon blocks that path, and the air travels down the trachea. This is the most common scenario when using the combitube.

page 35


The Fast Track

Spring 2014

If you don’t get breath sounds with tube one, ventilate through tube two. The second is the tracheal tube. If you get breath sounds, you have successfully blindly intubated the trachea. The device finding favor in prehospital and hospital circles it the King LT airway. This device combines the best of both of the other two devices. It has proximal and distal balloons like a combitube, but only one pilot balloon and one tube like an LMA. It is blindly inserted, the pilot balloon inflated and both proximal and distal balloons inflate. Ventilating through the King tube is similar to ventilating through the first tube of the combitube. The gold standard of airway management is placing a tube through the vocal cords. This is accomplished mostly through endotracheal intubation. Surgical airways, such as cricothyrotomy and tracheostomy accomplish the same thing (albeit bypassing the vocal cords), but that is beyond the scope of this article, as is a full discussion of intubation with the nuances of rapid sequence intubation, video laryngoscopy, and other such newer innovations of endotracheal intubation. Last but not least, airway management cannot be complete without as discussion on foreign body airway removal. The Heimlich maneuver is used by basic and advanced responders. Basic responders utilize a finger sweep to remove any dislodged objects; more advanced providers can look in with direct laryngoscopy and use specialized forceps to remove objects. Suction, either manual, battery operated, or vacuum operated is used to clear secretions, vomitus, blood or other liquids found in the airway. Many EMT’s will often put a conventional store-bought turkey baster in their personal first aid kits, which is essentially an adult version of the bulb syringe used in infants. Our next skill is needle thoracostomy, or needle decompression. This skill treats tension pneumothorax, which is fatal if untreated. Tension pneumothorax results from continued air leak from a simple pneumothorax now exerting increased air pressure on the heart, great vessels, and unaffected lung. Cardiac output falls, tidal volumes fall, and the patient dies if left untreated. The definitive treatment is tube thoracostomy, or chest tube placement. This is done in the hospital. Needle decompression can be done in the prehospital or hospital setting emergently. The needle is best placed in the second intercostal space, at the midclavicular line. The area should be quickly cleaned with an antiseptic such as alcohol, chlorexidine, or Betadine. A large bore IV catheter, most commonly a 14-gauge, is inserted above the second rib into the pleural space. The needle is removed and you should hear a rush of air. Commercial needle decompression kits are available, often with larger (10-gauge) needles and tubing to connect to a Heimlich, or flutter valve, to prevent air from being sucked into the chest. To do the same with a conventional IV catheter, cut the finger off an exam glove, tape it to the IV catheter hub, and make a slit in the fingertip of the glove. In pediatrics, smaller needles are used. Infants a 24-gauge can be used. Commercial kits offer another advantage: longer needles. Most standard IV catheters are 1.25 inches. You may need the 2.5 inch catheter, and, with obesity on the rise, even longer needles. An open pneumothorax can be treated with an occlusive dressing. This type of dressing prevents air from entering the chest but allows for escape. Classic teaching is to tape only three sides of the dressing in place, leaving one side open for air to escape. Vaseline impregnated gauze is often used, but there are commercial devices available that have built in flutter valves. Tension may still develop, and needle decompression may still be needed.

page 36

Our next skill is cardiac defibrillation. This is definitive treatment for ventricular fibrillation and pulseless ventricular tachycardia. It is the only thing proven to reverse VT/VF and is a class I recommendation from the American Heart


Association for cardiac arrest. In particular, it is earlier defibrillation. Defibrillation that happens later in the course of a cardiac arrest may not reverse the rhythm or worse, may break the rhythm leading to asystole. Automated external defibrillators (AED’s) have brought early defibrillation not only to BLS, but to the lay public. AED’s are now found in many public venues such as airports and shopping malls. It is often easier to perform CPR with an AED than without: simply turn it on and follow the instructions. Most manual defibrillators which are used in hospitals also have an AED feature. Our next skill is tourniquets. EMS has come full circle with tourniquet use. Removed from the EMS curriculum in the 1980’s for fear of resulting in limb amputations, our experience in the first Gulf War and the Global War on Terror has resulted in many more lives saved, and not necessarily an increase in limb amputations associated with tourniquets. Tourniquets should be used when direct pressure when elevation fails to control bleeding. They are considered first line in the treatment of exsanguinating hemorrhage and are one of the few skills that a triage officer can use when triaging patients at a mass casualty incident. All tourniquets should operate on a windlass, ratchet, or pneumatic principle. Many commercial windlass or ratchet style tourniquets are available. To make your own tourniquet, it must be wide (2-4 inches). Anything less than that could result in local tissue death from the pressure created by the narrow device. Cravats are often used to make a tourniquet, along with some type of stick to create the windlass. A Yankauer suction catheter accomplishes this. Place the cravat proximal to the wound, tie a simple knot, place the stick over the knot and tie a second knot over the stick. Twist the stick, tightening the cravat to the point where bleeding stops. If there is enough, tie the stick with the free ends of your cravat, or use a second cravat. Make sure to dress the wound now that the bleeding is stopped. Pneumatic tourniquets are used every day in assessment: the blood pressure cuff. To use the BP Cuff as a tourniquet, inflate the cuff to the point where bleeding stops, then clamp the tubing so air does not leak out of the cuff.

Many other commercial devices are available for bleeding control, especially for areas where tourniquets cannot be placed (shoulder, groin, neck, and abdomen). Hemostatic dressings are commonly used in these areas. Hemostatic dressings contain prothrombotic chemicals. Pressure dressings such as the Israeli bandage can also be used.

Spring 2014

The Fast Track

Our final skill is use of an epinephrine auto injector. This is the definitive treatment for anaphylaxis. Anaphylaxis is best defined as an allergic reaction affecting two or more body systems: of skin and respiratory, integumentary and circulatory, circulatory and respiratory could be the two, for example. Classic anaphylactic shock is hypotension following exposure to a known allergen. Epinephrine is warranted in both scenarios. The classic device is the Epi-Pen. There is now a generic epinephrine auto injector that works on the same principle: a safety cap on the back of the device is removed, arming the pen. The pen is the pushed into the lateral thigh, halfway between the hip and the knee. Firm pressure activates the device, and a spring loaded mechanism pushes the needle into the thigh and delivers the medicine. A newer device, Auvi-Q, is about the size of a flip phone. When removed from the carrying case, a voice prompt takes you through the steps of administration. A novel device which has not found much success on the market and may soon cease to exist is the Twin-Ject. This device has an auto injector function like the Epi-Pen, but also has a second dose that can be accessed by taking the device apart and removing the needle and syringe system from the auto injector housing, and a second dose can be given if needed via conventional intramuscular injection. As you can see, these interventions are for the most part, relatively simple. Airway management becomes complex with increased training, but the basics will get you out of trouble many times. Needle decompression can be accomplished quickly and can be taught quickly. It is found in the Combat Lifesaver courses taught to military personnel, as is the tourniquet. AED’s have revolutionized the treatment of VT/VF, as has the epinephrine auto injector for anaphylaxis. All of these skills are simple, easy to teach and are all, as you have seen, life saving.+

page 37


The Fast Track

Spring 2014

The View from the Bottom: A look at how things really were and are..... By Brian Lehnhof, DO PGY-2 @ Kent Hospital

W

ith the publishing of this article comes the end of intern year. Life as an intern is not always full of pleasurable experiences, however, it is filled with memorable lessons. Internship subjects us to things that we would have otherwise avoided. Through this process we interact with people we would not have met, and we learn lessons that we did not know existed. It is the sum of these experiences that create the foundation that we build the remainder of our medical career upon.

The long hours and scut work that seem to be the hallmark of intern year do not come as a surprise to most, but there are several things that did come as surprises to my fellow interns and me. My goal is to share some of the insights that would have been helpful to know early in the intern year. Day 1 is crucial: There will never be a time when you are more critically observed than in your first weeks of residency. Your performance on July 1 will be remembered for the next 4 years and possibly longer. The ED staff knows you are coming and if your program is like mine, your picture was probably distributed months ago. Be the person on day one that you want to be known as for the rest of residency. This does not mean you cannot make mistakes. You will make mistakes. The crucial factor is how you conduct yourself and the way you approach your role as a resident, not necessarily your medical knowledge. Take time to learn: The volume of patients coming through the ED can be intimidating as an intern. It can be difficult to take time to work on procedures such as placing central lines, doing ultrasounds or other such time consuming aspects of our training that can be seemingly deferred to someone else with more time or skill. This is particularly an issue when we as interns might not be proficient in these procedures, which further intensifies the time dilemma. Remember that this is your training and there is no better time to practice these procedures. Now is when you develop the skills and habits you will carry through your career. Make every effort to put your education at the forefront of the priorities. Your attendings should be willing to facilitate this by addressing the needs of other patients in the department.

page 38

Admit the truth: It is intimidating to admit you don’t understand a pathology or that you are weak in a procedure. However, nobody can know to assist you if you do not convey what you need. If you express your needs, others they will go out of their way to help with whatever the


issue may be. The expectations of you will never be lower than where it is today. Each day more is expected of us, so get the help you need today. I preface many of my questions with: “This may be a dumb question, but it I figure it is better to ask now, rather than as a 4th year.” Admit with confidence: There will be occasions where your disposition for patient will not align with that of your admitting colleague. In these settings we have to advocate for what is best for our patient. Nobody knows the patient better than us at that moment. It is tempting for an admitting physician to resist an admission when the presentation is weak or if there are deficits in the workup. As an intern you are easy prey. Be strong. If they sense fear, they will tear you and your case apart. You may not have to present all the details of the case, but know them in case they are needed. If you start your career with weak admissions, you will lose credibility with your admitting team and you will encounter resistance with admitting even the most straightforward of cases. Rehearse the presentation, even practice with your attending if needed, it will payoff in the end.

Spring 2014

The Fast Track

Own the ED: I did not fully appreciate the role that I would have as a resident in a training hospital. You are welcomed in a department full of facilitating attendings, but ultimately it is you and your fellow residents that will make the magic happen in the ED. You will spend more time in the ED than you will in your own home. The ED will become your home. Make it yours, have a personal investment in what goes on. Be active in the management of the ED. Take time to learn the names of your staff and build their confidence and respect in you. In many cases they could do your job better than you can; respect that. Off-service rotations are your key to the hospital: Not many EM residents await the day when they can round in the medicine wards or scrub in on an Proctosigmoidectomy. Unfortunately, there is no way to avoid some of the seemingly dreadful, boring, or painful off-service rotations. These off-service rotations are your chance to build relationships with your consultants and colleagues. The relationships you build while working shoulder to shoulder with your Medicine, Surgery, and Cardiology colleagues will make your job immensely easier down the road. Prove yourself to a specialist while on service with them, and you will have a far easier time admitting a patient to them or consulting them down the road at 3:00 am. In like manner if you burn a bridge with a specialist, your life is going to be miserable for the rest of residency. Be prepared to be humbled: A fellow intern said, “I wish I knew how uncomfortable, but ultimately rewarding it is to be an intern. The beginning of intern year makes it painfully clear that even though we learn a lot of informational knowledge in medical school, there is still a huge amount to learn about actually practicing medicine.” Intern year fosters exponential growth of both knowledge and character. Skills are developed and abilities are enhanced. Some of the things that seemed impossible last July are now daily routines. So much of what one learns as an intern is simply through the process and experience of being “the intern.” It will likely be one of the most uncomfortable years of medical training, but it can certainly been one of the most rewarding. To those embarking on their journey as an intern, I say to you: enjoy the ride. Brace yourself for all that is to come. Set your expectations high and pay the price to reach them. To those that have already traversed this path of internship, I say: congratulations but the journey is not over. Continue to fight the good fight that is medical training. This process is life long, but so also is the fulfillment it brings to us and to the patients we serve. +

page 39


The Fast Track

Spring 2014

Residency Spotlight

Ohio Valley Medical Center – Wheeling, West Virginia– Size: 16 EM residents (4 per year) 12 IM residents 4 EM/IM residents Total ED visits per year: 54,000 Hospital size: Level II trauma with 200 beds page 40


What is unique about your program? We have a great simulation lab with several different “bots’ and limbs, including the pregnant mom and pedibot. We work most shifts at OVMC, but up to five shifts/month may be at East Ohio Regional Hospital (20,000 ED visits/year). There is a drastic difference between the two patient populations. OVMC is an urban setting g and EORH is community, mostly insured patients.

hiking and camping. There’s also plenty of shopping, bars, restaurants in the Wheeling area. It’s also fun to go to a city for a night out: Pittsburgh is about one hour away and Columbus is about two hours away. Residents attend Pittsburgh Penguin and Pittsburgh Pirates games frequently. We also support our Wheeling Nailers!

Pittsburgh is one hour away and Columbus is about two hours away!

As residents, we are the “team doctor” for the Wheeling Nailers (ice hockey) and get to attend the games. We also run the medical tent at the annual ann al Jamboree in the Hills concert series, Tough Mudder, and the Ogden Race in Wheeling. As a group, we volunteer to perform sports physicals for St. Clairsville High School and Middle school.

Spring 2014

The Fast Track

Describe D the p r program in three w words: Hard-working, F Fun, Supportive

What is there to do outside of work? There’s so much to do when not working! Being in WV, there’s lots of rivers and lakes for swimming/boating/kayaking/white water rafting. g There’s several trails for

page 41


The Fast Track

Spring 2014

Incorporation of Spirituality in the Treatment of Emergency Medicine Patients By G Geoff eoffry ffry B B.. Comp, Comp, O OMS MS-IV IIV V @ KC KCUM KCUMB UMB B

Abstract Studies have demonstrated the importance of inclusion of spirituality and religion in multiple aspects of patients receiving medical care. However, there is a lack of consistency and emphasis on instruction of these subjects in the training of Emergency Medicine physicians. This article first examines the importance of the incorporation of spirituality in Emergency Medicine, as well as a brief review of current medical education standards. Finally, possible solutions are proposed involving various levels of medical education.

page 42

Introduction Physicians have many tools to assist in the healing process. They can use CT or MRI to “look inside” a patient, and use their eyes and ears to give additional diagnostic information. However, Emergency Medicine physicians have a specific challenge when implementing the Osteopathic, “whole person” approach when examining and interviewing patients. Patient interviews in the Emergency Department are more “problem-focused”, and lack some of the key features found in the traditional history and physical model that is utilized by many other medical specialties. “The interview is directed at actively considering the common and important (e.g., life threatening) etiologies for the problem at hand.”2 The often-brief face-to-face patient interaction requires quick thinking to initiate various levels of diagnosis and care, and may neglect the cultural, spiritual, or religious preferences of their patients. According to Douglas A Propp MD Chair, Department of Emergency Medicine at Advocate Lutheran General Hospital, Park

Ridge, IL, Emergency Medicine, “requires efficiency of thought, interaction, and time, which threatens our ability to invest in relationship building, the ‘‘spiritual currency’’ that many patients expect and deserve.”7 Ultimately, the most important way to ensure proper incorporation is to promote early communication and cultural competency education in medical school curriculum, and promote ongoing education regarding this subject. Issues The importance of including acknowledgement of patient spirituality, and possible integration in treatment has been well documented in literature. Paul S Mueller MD concludes: “A large and growing number of studies have shown a direct relationship between religious involvement and spirituality and positive health outcomes, including mortality, physical illnesses, mental illness, Healthcare Related Quality of Life, and coping with illness (including terminal illness.)” 6 Additional studies have shown that more patients desire conversations about religious and spiritual concerns than actually experience such interactions, and addressing this need may improve patient satisfaction with the medical care they receive.8,4 However, because of the nature of emergency medicine, this critical aspect of patient health may be overlooked. Not only does the determination of a patient’s spirituality help with outcome, it may also provide insight into the patient’s condition. Many different religions and cultures promote the use


of complementary and alternative medicine (CAM) in addition to traditional medical care, and its practice is becoming more prevalent. However, there is a discrepancy between patients’ stated comfort with disclosing their alternative medicine usage to their health care providers and actual disclosure, especially to ED physicians and nurses. It is important for physicians and nursing staff in Emergency Departments to consider questioning their patients about the use of CAM.3 The addition of this type of information may be vital in the appropriate treatment of the patient’s chief complaint and ultimate diagnosis. It is important for a physician to be aware of practices that may reflect a patient’s religious and spiritual beliefs, and be comfortable and confident when discussing them as the information gathered may add clues to aid in diagnosis.

instruction through the third and fourth year would allow for the incorporation of real-time clinical experiences and scenarios into the educational process.5

Spring 2014

The Fast Track

After graduation from medical school, Emergency Medicine residency programs could continue to build on the initial foundation. There is a high level of interest in creating a curriculum reflecting diversity and cultural competency in Emergency Medicine.1 However, barriers such as lack of time, and a shortage of qualified educational experts prevent full spectrum cultural and spiritual educational opportunities in many residency programs.1 A larger emphasis placed on instruction in residency programs would provide specific solutions to the potential lack of spirituality discussion in emergency room visits. Residents would be able to practice interview While there are clear benefits to the incor- techniques in real-world situations, and conporation of spirituality or belief systems into tinue to hone their understanding of various patient interviews and ultimate medical care, cultures. there is a significant deficit in spirituality education and related communication tech- An additional solution is to encourage discusniques in medical school curriculum. Both sion of spirituality and religion outside of a M.D. and D.O. medical schools have started to strict academic setting. Personal and student offer spirituality instruction, however, there is driven exploration of the topic should be ennot a major emphasis or global implementa- couraged as well. One technique is to support tion of any type of educational standards. A the development of medical students personstudy found that in 2004, 84 out of 126 ac- al belief systems. If students are comfortable credited medical schools offer courses in spir- with their own spirituality, they may be more ituality in medicine. Additionally, only 55% of inclined to discuss issues of culture and reall Osteopathic COMs in 2007 offered some ligion with their patients. Implementation of form of Spirituality in medicine instruction.5 scenario workshops at emergency medicine While this is an increase from previous years, conferences in both student and professional there are still numerous programs that do not capacities, allowing for discussion as to how offer this type of education. patient spirituality can be better implemented in the emergency room setting. Discussion A possible solution is to start spirituality in- The goal of a student’s religious and spiritual struction early in the medical education pro- study should be to better understand their cess, add additional instruction throughout all patient, and gain comfort in asking questions four years, and encourage continuing edu- regarding this topic in the patient interview. cation as a physician develops through their The focus of the curriculum should not be career. A majority of spirituality in medicine in-depth exploration of a patient’s system instruction occurs during the first two years of beliefs and religion, or to provide spiritual of medical school. An extension of spiritual guidance. It is important for physicians to recpage 43


The Fast Track

Spring 2014

ognize and acknowledge possible negative ethical implications in discussing these topics if not properly educated or experienced.6 However, it is entirely possible to address the spiritual needs of patients in a straightforward and non-controversial manner. Conclusion The importance of discussing spiritual and cultural issues with patients in the clinical setting has been widely discussed. However, due to the nature of emergency medicine there may be a deďŹ cit in the incorporation of this information into overall patient care. There is a lack of consistency in this type of education of Emergency Medicine physicians. The incorporation of spiritual, religious, and cultural competency education in multiple levels of medical education curriculums will ultimately improve patient health, satisfaction, and enhance the care provided by Emergency Medicine physicians. +

Student Chapter Highlights SATURDAY 10/11 Noon - Skills Lab 2pm - Rapid Fire Lectures SUNDAY 10/12 10am - Residency Fair 2pm - Student General Membership Meeting and Elections MONDAY 10/13 8am - Airway Shoot Out 2pm - Mock Interviews page 44


References (2003): 38-43. Pubmed. Web. 3 Jan. 2013. Drowning by Ben Abo, DO 1) Lawrence Ghislaine, ‘Tools of the Trade, Tobacco Smoke Enemas’ The Lancet vol. 359 issue. 9315 (April 2002): 1442. 2) Schmidt, Andrew Drowning Resuscitation 2014, accessed by internet May 24, 2014. 3) Vanden Hoek TL, Morrison LJ, Shuster M, Donnino M, Sinz E, Lavonas EJ, et al. Part 12: Cardiac arrest in special situations: 2010 American Heart Association guidelines for cardiopulmonary resuscitation and emergency cardiovascular care. Circulation. 2010;122:S829-61. 4) Szpilman, David. Near-drowning and drowning classification: A proposal to stratify mortality. Chest. 1997;112(3):660665. Pain Posters by Judd Shelton, OMSIV 1) Girion, L., & Glover, S. Doctors are top source of prescription drugs for chronic abusers. Los Angeles Times. March 3, 2014 http://www.latimes.com/local/la-me-rxsource-20140304,0,3275352.story. Accessed March 4, 2014 2)Klauer, K. ED Waiting Room Posters on Prescribing Pain Medications May Violate EMTALA. ACEP Now. January 8, 2014 http://www.acepnow.com/article/ed-waiting-roomposters-prescribing-pain-medications-may-violate-emtala/. Accessed March 4, 2014 3) Legal Information Institute. 42 U.S. Code § 1395dd - Examination and treatment for emergency medical conditions and women in labor. Cornell University Law School. http:// www.law.cornell.edu/uscode/text/42/1395dd. Accessed March 4, 2014 Pimpology by Danielle Turrin, MS, DO http://www.drmassoomi.com http://culpepperdds.files.wordpress.com/2011/01/anatomy-ofa-tooth.jpg http://emedicine.medscape.com/article/82755-overview http://en.wikipedia.org/wiki/Dentin http://en.wikipedia.org/wiki/Enamel Spirituality in the Emergency Department by Geoffrey B. Comp, OMSIV 1. Bowman, S. et al.. “Diversity in Emergency Medicine Education: Expanding the Horizon.” Academic Emergency Medicine 18.10 (2011): S104-109. PubMed. Web. 4 Jan. 2013. 2. “Emergency Medicine Clinical Clerkship Syllabus.” Http:// www.hsc.unt.edu/departments/clined/documents/Emergency%20Medicine%20Rotation.pdf. University of North Texas Health Science Center at Fort Worth, 3 Dec. 2011. Web. 12 Jan. 2103. 3. Kim, S. “A Multicenter Study of Complementary and Alternative Medicine Usage among ED Patients.” Academic Emergency Medicine 12.4 (2005): 377-80. PubMed. Web. 2 Jan. 2013. 4. MacLean, C. et al. “Patient Preference for Physician Discussion and Practice of Spirituality. Results From a Multicenter Patient Survey.” Journal of General Internal Medicine 18.1

5. McClain, E. et al. “Spirituality and Medicine: Prevalence of Spirituality-in-Medicine Instruction at Osteopathic Medical Schools.” JAOA 108.4 (2008): 197-202. PubMed. Web. 2 Jan. 2013. 6. Mueller, P. S., D. J. Plevak, and T. A. Rummans. “Religious Involvement, Spirituality, and Medicine: Implications for Clinical Practice.” Mayo Clinic Proceedings 76.12 (2001): 1225-235. PubMed. Web. 2 Jan. 2013. 7. Propp, Douglas A., MD. “Is Spirituality an Emergency Physician’s Competency?” Academic Emergency Medicine 10.10 (2003): 1098-099. PubMed. Web. 2 Jan. 2013. 8. Williams, J. et al. “Attention to Inpatients’ Religious and Spiritual Concerns: Predictors and Association with Patient Satisfaction.” Journal of General Internal Medicine 26.11 (2011): 1265-271. Pubmed. Web. 3 Jan. 2013.

Spring 2014

The Fast Track

Tricks of the Trade: Fractures in the Wild by Chase Ungs, DO 1) Menkes, J. Initial Evaluation and Management of Orthopedic Injuries. In: Tintinalli JE, Kelen GD, Stapczynski JS, Ma, OJ, Cline DM, editors. Tintinalli’s Emergency Medicine. 6thed. New York: McGraw-Hill; 2011. Accesses May 26, 2014 from http://accessemergencymedicine.mhmedical.com/book. aspx?booki d=693 2) Plantz, Scott. “Fractures or Dislocations Causes, Symptoms, Treatment - Fractures or Dislocations Treatment - EMedicineHealth.” EMedicineHealth. Ed. N. Harris, Francisco Talavera, and James Takayesu. N.p., 10 Aug. 2005. Web. 28 May 2014.

Snake Bites by Nate Moore, OMSIII 1) Barish, R., & Arnold, T. (2013, May 1). Snakebites. Merck Manuals Professional. Retrieved April 1, 2014, from http:// www.merckmanuals.com/professional/injuries_poisoning/ bites_and_stings/snakebites.html?qt=snake%20&alt=sh 2) Blanc P.D. (2012). Chapter 240. Evaluation of the Patient with Occupational Chemical Exposure. In Olson K.R. (Eds), Poisoning & Drug Overdose, 6e.Retrieved April 26, 2014 fromhttp://accessmedicine.mhmedical.com/content.aspx?boo kid=391&Sectionid=42070056 3) Cheng C, et al. Management of Crotalinae Bites in the United States, In: UpToDate, Post TW (Ed), UpToDate, Waltham, MA. (Accessed on 4/21/14.) 4) Cheng, C, et al. Management of Elapidae Bites in the United States, In: UpToDate, Post TW (Ed), UpToDate, Waltham, MA. (Accessed on 4/21/14.) 5) Juckett, G , Hancox, J. West Virginia University School of Medicine, Morgantown, West Virginia.Am Fam Physician. 2002 Apr 1;65(7):1367-1375. 6) Otten EJ. Venomous animal injuries. In: Marx JA, ed. Rosen’s Emergency Medicine: Concepts and Clinical Practice. 7th ed. Philadelphia, Pa: Mosby Elsevier; 2009:chap 59. 7) Seifert SA, et al. AAPCC database characterization of native US venomous snale exposures. 2001-2005. Clinical Toxicology (Philadelphia) 2009, 47:327 page 45


The Fast Track

Review Answers

Spring 2014

Question 1: Answer B. This patient is suffering from croup, or laryngotracheobronchitis. Croup is a viral infection of the upper airway, most commonly caused by parainfluenza virus. It is the most common cause of upper airway distress and obstruction in children. The peak incidence of croup is 2 years and the classic age range of children affected is 6 months to 6 years. Most cases occur in late fall, winter and early spring. Patients have a characteristic barking cough that sounds like a seal. Symptoms occur because of edema and exudate of the subglottic space. Croup is a clinical diagnosis based on the presence of the barking cough, hoarse voice and sometimes stridor. Preceding the development of these symptoms, children may have had a prodrome of fever and URI symptoms. The administration of dexamethasone is the mainstay of therapy and has been shown to decrease the duration of symptoms, decrease return visits to the ED, decrease length of stay in the ED and decrease the need for epinephrine. In patients with stridor, the administration of racemic epinephrine leads to rapid improvement of symptoms in most cases. Patients who have received racemic epinephrine need a period of observation after treatment for 1-2 hours to ensure that stridor and retractions do not recur. On X-ray, the steeple sign is present due to a narrowing of the upper trachea from the infection. Albuterol (A) is a beta-agonist used in the treatment of bronchoconstriction. Beta-agonists have no benefit in the treatment of patients with croup, as this is a disease of the upper airway. Racemic epinephrine (C) is typically reserved for patients with significant or resting stridor or signs of respiratory distress with retractions. After the administration of nebulized racemic epinephrine, patients must be observed for recurrent symptoms. Ribavirin (D) is an anti-viral medication sometimes used in the management of hepatitis C, but also is used off-label in the treatment of some respiratory syncytial virus (RSV) infections. Ref: Cukor J, Manno M: Pediatric Respiratory Emergencies: Upper Airway Obstruction and Infections, in Marx JA, Hockberger RS, Walls RM, et al (eds): Rosen’s Emergency Medicine: Concepts and Clinical Practice, ed 8. St. Louis, Mosby, Inc., 2013, (Ch) 168: pp 2111-2114. Question 2: Answer: C. This patient presents with epiglottitis. Epiglottitis has become less common in children since the development of the H. influenzae vaccine. Classically these children are toxic appearing with significant pain in the throat or neck or both. In severe cases, patients will sit in the sniffing position, often “tripoding” in order to maintain an adequate airway. Given the degree of epiglottal inflammation, patients are at high risk for airway obstruction and all efforts to avoid agitating the patient should be attempted. Since the illness is rapidly progressive, patients will often need definitive airway management. In order to maximize the success of this, management in the operating room with resources and a controlled environment is indicated. Additionally, surgeons are present if the need arises for a surgical airway. In addition to airway management, patients need antibiotic therapy, most commonly ceftriaxone.

The insertion of two intravenous lines (A) will likely agitate this child. With increased crying, additional compromise of the tenuous airway will take place and should be avoided at all costs. Access is important in this life-threatening emergency, but optimizing insertion is important (in the OR if time permits or one line to begin). Although it may be helpful to identify swelling in the posterior pharynx, inspection of the posterior pharynx using a tongue depressor (B) is not indicated. This will also agitate the child and further exacerbate the already compromised airway. Racemic epinephrine nebulizers (D) are indicated in cases of croup where tracheal edema causes stridor. Racemic epinephrine is not routine for the management of epiglottitis and page again a mask over the child’s face may upset him and cause additional difficulty breathing. 46


Ref: Melio FR, Berge LR: Upper Respiratory Tract Infections, in Marx JA, Hockberger RS, Walls RM, et al (eds): Rosen’s Emergency Medicine: Concepts and Clinical Practice, ed 8. St. Louis, Mosby, Inc., 2013, (Ch) 75: pp 969-971.

Spring 2014

The Fast Track

Question 3: Answer: A. The PECARN network has provided the largest study ever done on pediatric head trauma now guides the decision-making for CT scan in head trauma. In children under the age of two, with a GCS score of 14 or higher, the presence of parietal, occipital or temporal scalp hematomas is predictive of intracranial injury. Frontal or forehead hematomas were not predictive of intracranial injury. Other indications for observation or CT scan include: history of loss of consciousness for at least 5 seconds, severe mechanism of injury, or not acting normally per the parents. The presence of any of these characteristics (including the scalp hematoma) has a 0.9% risk of traumatic brain injury. The PECARN decision rule is 100% sensitive in children under 2. Occipital (B), parietal (C) and temporal (D) bone hematomas doe not predict traumatic brain injury.

Ref: Kuppermann N, Holmes JF, Dayan PS, Hoyle JD Jr, Atabaki SM, Holubkov R, Nadel FM, Monroe D, Stanley RM, Borgialli DA, Badawy MK, Schunk JE, Quayle KS, Mahajan P, Lichenstein R, Lillis KA, Tunik MG, Jacobs ES, Callahan JM, Gorelick MH, Glass TF, Lee LK, Bachman MC, Cooper A, Powell EC, Gerardi MJ, Melville KA, Muizelaar JP, Wisner DH, Zuspan SJ, Dean JM, Wootton-Gorges SL; Pediatric Emergency Care Applied Research Network (PECARN). Identification of children at very low risk of clinically-important brain injuries after head trauma: a prospective cohort study. Lancet. 2009 Oct 3;374(9696):1160-70. Epub 2009 Sep 14. PubMed PMID: 19758692. Question 4: Answer: D. The EMT-P (emergency medical technician-paramedic) is the most advanced provider in the EMS system. Training includes classroom didactics, clinical rotations and field experience. Depending on the model which varies by state, certification may run from one to four years and in the four year model include a bachelor’s degree. EMT-P’s have a wide scope of practice that includes cardiac rhythm analysis, pharmacologic treatments and advanced airway techniques. AEMTs (A) are advanced EMTs and previously were called EMT-I (intermediate). AEMTs have a wider scope of practice than a basic EMT but do not achieve the level of paramedic. The certification was designed for regions in which paramedic services were unavailable. The exact scope of practice varies by state certification but typically includes some advanced life support services. An EMD (B) is an emergency medical dispatcher. EMDs work in a communication center and answer 911 calls for medical assistance. They determine the level of response needed for given emergencies (Basic Life Support of Advanced Life Support). For some life threatening conditions, the EMD also provides guidance to the caller for critical interventions (e.g. CPR). An EMT (C) is the minimal level required to staff an ambulance and provide care to patients. In addition to basic first aid, EMTs are trained in defibrillation, epinephrine autoinjection, albuterol administration and the use of some airway adjuncts. Ref: Blackwell TH. Emergency Medical Services: Overview and Ground Transport, in Marx JA, Hockberger RS, Walls RM, et al (eds): Rosen’s Emergency Medicine: Concepts and Clinical Practice, ed 7. St. Louis, Mosby, Inc., 2013, (Ch) 190: pp 1918-1919.

page 47


The Fast Track

Keep an eye out for our next issue! Spring 2014

ACOEP Resident and Student Chapter 142 East Ontario Street Suite 1500 Chicago, Illinois 60611 Phone: 312.587.3709 page Fax: 312.587.9951 48 E-mail: fasttrack@acoep.org

Follow us on

@ ACOEPSC @ ACOEPRC @ facebook.com/acoepsc @ facebook.com/acoeprc


Turn static files into dynamic content formats.

Create a flipbook
Issuu converts static files into: digital portfolios, online yearbooks, online catalogs, digital photo albums and more. Sign up and create your flipbook.